- Wed Sep 27, 2017 4:27 pm
#40157
Complete Question Explanation
(The complete setup for this game can be found here: lsat/viewtopic.php?t=8560)
The correct answer choice is (E)
This is the infamous Rule Substitution question, a staple in the Logic Games section as of late. Considering their growing popularity, tackling such questions is no longer merely optional: it is imperative to have a workable strategy in place.
The question stem asks you to replace the second rule of the game, according to which neither X nor Y can be earlier than Z:
Our job is to substitute that rule with a logically equivalent condition, which would have the exact same effect on the order in which the speeches are given. While the wording of the correct answer can be difficult-to-impossible to predict, the four incorrect answer choices will either present rules that were not part of the original rule set (also known as Additional Effects conditions), or else they will only partially constrain the variables in the rule being substituted (i.e. Partial Match conditions).
Since Additional Effects are easier to spot, look for answer choices that contain such conditions first. You should immediately notice, for instance, that answer choice (A) adds a condition extraneous to the original rule set, because it stipulates that L must be at 3 PM. This is not required by the original rule set, given that L can also be at 2 PM (see the local diagram for Question #10).
We are left with answer choices (B), (C), (D), and (E).
Answer choice (B): This rule prohibits M from being earlier than Z. In other words, M must be given either at the same time as Z, or else later than Z:
Clearly, answer choice (B) is a Partial Match condition, and is therefore incorrect.
Answer choice (C) is also incorrect. Here, the stipulation is that either X or Y be after Z. This is clearly an implication of the rule we need to substitute, because it is impossible for all three speeches to be given at the same time, and so at least one of X or Y would have to be given after Z. However, our job is to ensure that neither X nor Y can be earlier than Z, which this rule cannot do. For instance, the following solution would be possible if we are allowed to place just one of X or Y after Z:
Therefore, answer choice (C) also contains a Partial Match, and must be eliminated.
Answer choice (D) states that either X or Y (or both) must be at 2 PM; in other words, at least one of them must be at 2 PM. This is clearly true given the original rule set, because X and Y cannot both be given at 1 PM without violating the second rule, and there is only one speech being given at 3 PM. However, requiring just one of X or Y to be at 2 PM still allows the other one to be earlier than Z, in violation of the rule being substituted. For instance, the following hypothetical setup would be possible with this restraint in effect:
Thus, answer choice (D) also contains a Partial Match, and must be eliminated.
Answer choice (E) is the correct answer choice. The condition requiring Z to be at 1 PM is consistent with the original diagram, as it is the main inference in the game. Answer choice (E), therefore, does not contain an Additional Effects rule. The better question is: does answer choice (E) contain a Partial Match? It does not, because if Z is at 1 PM, then neither X nor Y can be earlier than Z. Thus, the effect of the new condition is identical to that of the original rule, and answer choice (E) is correct.
(The complete setup for this game can be found here: lsat/viewtopic.php?t=8560)
The correct answer choice is (E)
This is the infamous Rule Substitution question, a staple in the Logic Games section as of late. Considering their growing popularity, tackling such questions is no longer merely optional: it is imperative to have a workable strategy in place.
The question stem asks you to replace the second rule of the game, according to which neither X nor Y can be earlier than Z:
Our job is to substitute that rule with a logically equivalent condition, which would have the exact same effect on the order in which the speeches are given. While the wording of the correct answer can be difficult-to-impossible to predict, the four incorrect answer choices will either present rules that were not part of the original rule set (also known as Additional Effects conditions), or else they will only partially constrain the variables in the rule being substituted (i.e. Partial Match conditions).
Since Additional Effects are easier to spot, look for answer choices that contain such conditions first. You should immediately notice, for instance, that answer choice (A) adds a condition extraneous to the original rule set, because it stipulates that L must be at 3 PM. This is not required by the original rule set, given that L can also be at 2 PM (see the local diagram for Question #10).
We are left with answer choices (B), (C), (D), and (E).
Answer choice (B): This rule prohibits M from being earlier than Z. In other words, M must be given either at the same time as Z, or else later than Z:
- Z======M
Clearly, answer choice (B) is a Partial Match condition, and is therefore incorrect.
Answer choice (C) is also incorrect. Here, the stipulation is that either X or Y be after Z. This is clearly an implication of the rule we need to substitute, because it is impossible for all three speeches to be given at the same time, and so at least one of X or Y would have to be given after Z. However, our job is to ensure that neither X nor Y can be earlier than Z, which this rule cannot do. For instance, the following solution would be possible if we are allowed to place just one of X or Y after Z:
Therefore, answer choice (C) also contains a Partial Match, and must be eliminated.
Answer choice (D) states that either X or Y (or both) must be at 2 PM; in other words, at least one of them must be at 2 PM. This is clearly true given the original rule set, because X and Y cannot both be given at 1 PM without violating the second rule, and there is only one speech being given at 3 PM. However, requiring just one of X or Y to be at 2 PM still allows the other one to be earlier than Z, in violation of the rule being substituted. For instance, the following hypothetical setup would be possible with this restraint in effect:
Thus, answer choice (D) also contains a Partial Match, and must be eliminated.
Answer choice (E) is the correct answer choice. The condition requiring Z to be at 1 PM is consistent with the original diagram, as it is the main inference in the game. Answer choice (E), therefore, does not contain an Additional Effects rule. The better question is: does answer choice (E) contain a Partial Match? It does not, because if Z is at 1 PM, then neither X nor Y can be earlier than Z. Thus, the effect of the new condition is identical to that of the original rule, and answer choice (E) is correct.
You do not have the required permissions to view the files attached to this post.